Question

Which of the following statements is correct?: A.). When expected rate of return is greater than...

Which of the following statements is correct?:

A.). When expected rate of return is greater than required rate of return, this stock is overpriced.

B.). The Beta of treasury bill is 0.

C.). Systematic risk can be eliminated through diversification.

Homework Answers

Answer #1

Option (B) is correct

The Beta of treasury bill is 0.

Beta measures the volatility of the stock returns with the market returns. Treasury bills are risk free assets / rates. Their rates do not change. So, there is less or no volatility of their returns.

Option (A) is incorrect because when the expected rate of return is greater than required rate of return, then the stock is underpriced.

Option (C) is incorrect because it is unsystematic risk, an not systematic risk that can be eliminated through diversification.

Know the answer?
Your Answer:

Post as a guest

Your Name:

What's your source?

Earn Coins

Coins can be redeemed for fabulous gifts.

Not the answer you're looking for?
Ask your own homework help question
Similar Questions
Which of the following statements is most correct? The required rate of return of a diversified...
Which of the following statements is most correct? The required rate of return of a diversified portfolio with Beta of 1 is typically greater than the Market Risk Premium. A stock with a negrative beta must have a negative required rate of return. If a stock's beta doubles its required rate of return must double. If a stock has a beta equal to 1.0, its required rate of return will be unaffected by changes in the market risk premium. None...
2. Which of the following statements concerning beta is correct? a. A stock with a beta...
2. Which of the following statements concerning beta is correct? a. A stock with a beta of 0 is expected to provide a rate of return equal to the market portfolio b. A stock with a beta equal to 1 has no risk c. Stocks with negative betas have the least amount of risk FALSE d. A stock with a beta greater than 1 is expected to be more volatile than the market portfolio
Which of the following statements is false? Question 11 options: a The portfolio that contains all...
Which of the following statements is false? Question 11 options: a The portfolio that contains all shares of all stocks and securities in the market is called the efficient portfolio. b Systematic risk cannot be eliminated through diversification. c A portfolio that contains only systematic risk is called an efficient portfolio. d Volatility measures total risk, while beta measures only systematic risk. e None of the above.
QUESTION 17 Which of the following statements is most correct? a. An increase in expected inflation...
QUESTION 17 Which of the following statements is most correct? a. An increase in expected inflation could be expected to increase the required return on a riskless asset and on an average stock by the same amount, other things held constant. b. A graph of the SML would show required rates of return on the vertical axis and standard deviations of returns on the horizontal axis. c. If two "normal" or "typical" stocks were combined to form a 2-stock portfolio,...
Which of the following is true about diversification? a. The expected return on a risky asset...
Which of the following is true about diversification? a. The expected return on a risky asset depends on that asset’s unsystematic risk b. Portfolio diversification is the investment in several but same asset classes or sectors/industry c. Beta is a measure of systematic risk d. There is a large portion of unsystematic risk in a well diversified portfolio
Stock A's beta is 1.5 and Stock B's beta is 0.5. Which of the following statements...
Stock A's beta is 1.5 and Stock B's beta is 0.5. Which of the following statements must be true, assuming the CAPM is correct. a. Stock A would be a more desirable addition to a portfolio then Stock B. b. Stock B would be a more desirable addition to a portfolio than A. c. When held in isolation, Stock A has more risk than Stock B. d. In equilibrium, the expected return on Stock B will be greater than that...
Stock A's beta is 1.5 and Stock B's beta is 0.5. Which of the following statements...
Stock A's beta is 1.5 and Stock B's beta is 0.5. Which of the following statements must be true, assuming the CAPM is correct. a. Stock A would be a more desirable addition to a portfolio then Stock B. b. In equilibrium, the expected return on Stock B will be greater than that on Stock A. c. When held in isolation, Stock A has more risk than Stock B. d. Stock B would be a more desirable addition to a...
(5 marks) Which of the following statements are correct? a. Stock A has an expected return...
Which of the following statements are correct? a. Stock A has an expected return of 10% and a standard deviation of 15%, and stock B has an expected return of 13% and a standard deviation of 14%. No investor would ever buy stock A because it has a lower expected return and a higher risk than stock B. b. A firm is expected to pay a dividend of £3 per share in one year. This dividend is expected to grow...
Which of the following Statements is most accurate? The greater the number of stocks in a...
Which of the following Statements is most accurate? The greater the number of stocks in a stock portfolio, the harder it would be to outperform a market index like the S&P 500 The greater the number of stocks in a stock portfolio, the higher the beta of the portfolio The greater the number of stocks in a stock portfolio, the lower the portfolio's systematic risk The beta of a sock portfolio will increase any time that one or more of...
HW #6 1. Use the following information to answer the questions. State Probability Stock A return...
HW #6 1. Use the following information to answer the questions. State Probability Stock A return Stock B return Good Normal Bad 0.3 0.6 0.1 8% 2% -3% 5% 1% -1% (a). Given that you form a portfolio by investing $4,000 in Stock A and $1,000 in Stock B, what is the expected return on your portfolio? (b).What is the variance and standard deviation of your portfolio? (c). Suppose that Stock A has a beta of 1.5 and Stock B...